Triangle inequality for complex numbers: sketch of proof

Click For Summary
The discussion focuses on proving the triangle inequality for complex numbers, specifically that |z_1 + z_2| ≤ |z_1| + |z_2| for z_1, z_2 in the complex plane. One participant attempts to approach the proof through explicit calculations involving square roots and inequalities but expresses uncertainty about the validity of their method. Another contributor suggests using complex conjugates to simplify the proof, recommending starting from the expression |z_1 + z_2|^2 = (z_1 + z_2)(\bar{z_1} + \bar{z_2}). The conversation emphasizes the importance of proper mathematical techniques in establishing the inequality. The discussion ultimately seeks clarity on the correct approach to proving the triangle inequality in this context.
Advent
Messages
29
Reaction score
0

Homework Statement



Show that if z_1,z_2 \in \mathbb{C} then |z_1+z_2| \leq |z_1| + |z_2|

Homework Equations



Above.

The Attempt at a Solution



I tried by explicit calculation, with obvious notation for a,b and c: my frist claim is not that the triangle inequality holds, just that I don't know to put a ? above the \leq symbol

\sqrt{a} \leq \sqrt{b} + \sqrt{c} \rightarrow 0 \leq \sqrt{b} <br /> + \sqrt{c} - \sqrt{a} \rightarrow 0 \leq \frac{b+c+2\sqrt{b}\sqrt{c} - a}{\sqrt{b}+\sqrt{c}+\sqrt{a}}
Now if z_1 = x_1 + i y_1 and z_2=x_2+iy_2

and using again the conjugate of the roots expresion, the last equation is something like

0\leq f(x_1^2,x_2^2,y_1^2,y_2^2)

and so is true. can this be correct or may I write explicitly all the terms?

Thanks.
 
Physics news on Phys.org
I really don't think that's going anywhere. You should use complex conjugates to express the absolute values. |z|^2=z \bar{z}. Start from |z_1+z_2|^2=(z_1+z_2) (\bar{z_1} + \bar{z_2})
 
Question: A clock's minute hand has length 4 and its hour hand has length 3. What is the distance between the tips at the moment when it is increasing most rapidly?(Putnam Exam Question) Answer: Making assumption that both the hands moves at constant angular velocities, the answer is ## \sqrt{7} .## But don't you think this assumption is somewhat doubtful and wrong?

Similar threads

  • · Replies 6 ·
Replies
6
Views
2K
Replies
7
Views
4K
  • · Replies 2 ·
Replies
2
Views
2K
Replies
2
Views
1K
  • · Replies 2 ·
Replies
2
Views
1K
Replies
20
Views
3K
  • · Replies 16 ·
Replies
16
Views
4K
  • · Replies 9 ·
Replies
9
Views
2K
  • · Replies 11 ·
Replies
11
Views
2K
  • · Replies 3 ·
Replies
3
Views
2K